Mechanics - Elastic springs and strings

Click For Summary
A particle P of mass m is attached to an elastic string with a natural length L, and when in equilibrium, the length AP is \(\frac{5l}{3}\). If P is projected downwards with a speed of \(\sqrt{\frac{3gl}{2}}\), it will come to instantaneous rest after moving a distance of \(\frac{10l}{3}\). The discussion involves calculating kinetic energy (KE), gravitational potential energy (GPE), and elastic potential energy (EPE) before and after the projection. There is some confusion regarding the energy calculations, and the poster suggests that this might be better suited for a physics-focused forum. The conversation highlights the need for clarity in applying energy conservation principles in this context.
mr bob
Messages
38
Reaction score
0
A particle P of mass m is attached to one end of a light elastic string of natural length L whose other end is attached to a point A on a ceiling. When P hangs in equilibrium AP has length \frac{5l}{3}. Show that if P is projected vertically downwards from A with speed \sqrt(\frac{3gl}{2}), P will come to instantaneous rest after moving a distance \frac{10l}{3}.

I thought about working all this out by finding the energies before and after the projection.

Before:-
KE =\frac{3gl}{4}
GPE = 0
EPE = 0

After:-
KE = 0
GPE = -(y - 5/3L)g where y is the full length of stretched string.
EPE = \frac{gl(Y- 5/3L)^2}{2L(Y- 5/3L)}

i can't seem to get the anwer using these, are they correct?
 
Last edited:
Physics news on Phys.org
This seems like more of a physics related question. I'm sure if you post it in the physics section someone would help you out a little faster. I haven't taken differential equation in awhile, but this seems like a pretty straightforward differential equation question. I could definitely be wrong though
 
Question: A clock's minute hand has length 4 and its hour hand has length 3. What is the distance between the tips at the moment when it is increasing most rapidly?(Putnam Exam Question) Answer: Making assumption that both the hands moves at constant angular velocities, the answer is ## \sqrt{7} .## But don't you think this assumption is somewhat doubtful and wrong?

Similar threads

Replies
5
Views
2K
  • · Replies 19 ·
Replies
19
Views
4K
Replies
6
Views
8K
Replies
4
Views
4K
Replies
3
Views
1K
Replies
6
Views
1K
Replies
5
Views
2K
  • · Replies 1 ·
Replies
1
Views
2K
Replies
8
Views
2K
  • · Replies 12 ·
Replies
12
Views
3K